2 Poisson distributions time distribution given one of them occurs first












0












$begingroup$


Suppose that A and B are two independent Poisson distribution with parameter $lambda_a$ and $lambda_b$ denoting the number of, say the received emails per hour in two different computers. B is active after an hour later $t>1$. I want to find the expected value of the time that A receives the mail for the first time given A does not receive anything in the interval $t=[0,1]$ and before B does.



My attempt:



I use the same technique as the one Poisson rv as follows



The time distribution minus one is just($x =t-1$)



$P(X leq x) = $P("B does not receive anything but A receives at least one" ) = $(1-e^{-lambda_a x})e^{-lambda_b x} $



Based on this we can find the mean which is $frac{1}{lambda_b + lambda_a}- frac{1}{lambda_b} $ . But the correct answer is same as this except the second term takes positive sign. Is there any reasoning error here? If yes how to resolve this?










share|cite|improve this question











$endgroup$








  • 1




    $begingroup$
    It's quite difficult for me to parse your question. Could you please clarify it (rewrite parts of it)?
    $endgroup$
    – zoidberg
    Dec 9 '18 at 5:28
















0












$begingroup$


Suppose that A and B are two independent Poisson distribution with parameter $lambda_a$ and $lambda_b$ denoting the number of, say the received emails per hour in two different computers. B is active after an hour later $t>1$. I want to find the expected value of the time that A receives the mail for the first time given A does not receive anything in the interval $t=[0,1]$ and before B does.



My attempt:



I use the same technique as the one Poisson rv as follows



The time distribution minus one is just($x =t-1$)



$P(X leq x) = $P("B does not receive anything but A receives at least one" ) = $(1-e^{-lambda_a x})e^{-lambda_b x} $



Based on this we can find the mean which is $frac{1}{lambda_b + lambda_a}- frac{1}{lambda_b} $ . But the correct answer is same as this except the second term takes positive sign. Is there any reasoning error here? If yes how to resolve this?










share|cite|improve this question











$endgroup$








  • 1




    $begingroup$
    It's quite difficult for me to parse your question. Could you please clarify it (rewrite parts of it)?
    $endgroup$
    – zoidberg
    Dec 9 '18 at 5:28














0












0








0





$begingroup$


Suppose that A and B are two independent Poisson distribution with parameter $lambda_a$ and $lambda_b$ denoting the number of, say the received emails per hour in two different computers. B is active after an hour later $t>1$. I want to find the expected value of the time that A receives the mail for the first time given A does not receive anything in the interval $t=[0,1]$ and before B does.



My attempt:



I use the same technique as the one Poisson rv as follows



The time distribution minus one is just($x =t-1$)



$P(X leq x) = $P("B does not receive anything but A receives at least one" ) = $(1-e^{-lambda_a x})e^{-lambda_b x} $



Based on this we can find the mean which is $frac{1}{lambda_b + lambda_a}- frac{1}{lambda_b} $ . But the correct answer is same as this except the second term takes positive sign. Is there any reasoning error here? If yes how to resolve this?










share|cite|improve this question











$endgroup$




Suppose that A and B are two independent Poisson distribution with parameter $lambda_a$ and $lambda_b$ denoting the number of, say the received emails per hour in two different computers. B is active after an hour later $t>1$. I want to find the expected value of the time that A receives the mail for the first time given A does not receive anything in the interval $t=[0,1]$ and before B does.



My attempt:



I use the same technique as the one Poisson rv as follows



The time distribution minus one is just($x =t-1$)



$P(X leq x) = $P("B does not receive anything but A receives at least one" ) = $(1-e^{-lambda_a x})e^{-lambda_b x} $



Based on this we can find the mean which is $frac{1}{lambda_b + lambda_a}- frac{1}{lambda_b} $ . But the correct answer is same as this except the second term takes positive sign. Is there any reasoning error here? If yes how to resolve this?







poisson-process exponential-distribution






share|cite|improve this question















share|cite|improve this question













share|cite|improve this question




share|cite|improve this question








edited Dec 9 '18 at 6:15







Interception

















asked Dec 9 '18 at 5:18









InterceptionInterception

233




233








  • 1




    $begingroup$
    It's quite difficult for me to parse your question. Could you please clarify it (rewrite parts of it)?
    $endgroup$
    – zoidberg
    Dec 9 '18 at 5:28














  • 1




    $begingroup$
    It's quite difficult for me to parse your question. Could you please clarify it (rewrite parts of it)?
    $endgroup$
    – zoidberg
    Dec 9 '18 at 5:28








1




1




$begingroup$
It's quite difficult for me to parse your question. Could you please clarify it (rewrite parts of it)?
$endgroup$
– zoidberg
Dec 9 '18 at 5:28




$begingroup$
It's quite difficult for me to parse your question. Could you please clarify it (rewrite parts of it)?
$endgroup$
– zoidberg
Dec 9 '18 at 5:28










0






active

oldest

votes











Your Answer





StackExchange.ifUsing("editor", function () {
return StackExchange.using("mathjaxEditing", function () {
StackExchange.MarkdownEditor.creationCallbacks.add(function (editor, postfix) {
StackExchange.mathjaxEditing.prepareWmdForMathJax(editor, postfix, [["$", "$"], ["\\(","\\)"]]);
});
});
}, "mathjax-editing");

StackExchange.ready(function() {
var channelOptions = {
tags: "".split(" "),
id: "69"
};
initTagRenderer("".split(" "), "".split(" "), channelOptions);

StackExchange.using("externalEditor", function() {
// Have to fire editor after snippets, if snippets enabled
if (StackExchange.settings.snippets.snippetsEnabled) {
StackExchange.using("snippets", function() {
createEditor();
});
}
else {
createEditor();
}
});

function createEditor() {
StackExchange.prepareEditor({
heartbeatType: 'answer',
autoActivateHeartbeat: false,
convertImagesToLinks: true,
noModals: true,
showLowRepImageUploadWarning: true,
reputationToPostImages: 10,
bindNavPrevention: true,
postfix: "",
imageUploader: {
brandingHtml: "Powered by u003ca class="icon-imgur-white" href="https://imgur.com/"u003eu003c/au003e",
contentPolicyHtml: "User contributions licensed under u003ca href="https://creativecommons.org/licenses/by-sa/3.0/"u003ecc by-sa 3.0 with attribution requiredu003c/au003e u003ca href="https://stackoverflow.com/legal/content-policy"u003e(content policy)u003c/au003e",
allowUrls: true
},
noCode: true, onDemand: true,
discardSelector: ".discard-answer"
,immediatelyShowMarkdownHelp:true
});


}
});














draft saved

draft discarded


















StackExchange.ready(
function () {
StackExchange.openid.initPostLogin('.new-post-login', 'https%3a%2f%2fmath.stackexchange.com%2fquestions%2f3032038%2f2-poisson-distributions-time-distribution-given-one-of-them-occurs-first%23new-answer', 'question_page');
}
);

Post as a guest















Required, but never shown

























0






active

oldest

votes








0






active

oldest

votes









active

oldest

votes






active

oldest

votes
















draft saved

draft discarded




















































Thanks for contributing an answer to Mathematics Stack Exchange!


  • Please be sure to answer the question. Provide details and share your research!

But avoid



  • Asking for help, clarification, or responding to other answers.

  • Making statements based on opinion; back them up with references or personal experience.


Use MathJax to format equations. MathJax reference.


To learn more, see our tips on writing great answers.




draft saved


draft discarded














StackExchange.ready(
function () {
StackExchange.openid.initPostLogin('.new-post-login', 'https%3a%2f%2fmath.stackexchange.com%2fquestions%2f3032038%2f2-poisson-distributions-time-distribution-given-one-of-them-occurs-first%23new-answer', 'question_page');
}
);

Post as a guest















Required, but never shown





















































Required, but never shown














Required, but never shown












Required, but never shown







Required, but never shown

































Required, but never shown














Required, but never shown












Required, but never shown







Required, but never shown







Popular posts from this blog

How do I know what Microsoft account the skydrive app is syncing to?

When does type information flow backwards in C++?

Grease: Live!